Đến nội dung

Nguyen Tang Sy nội dung

Có 54 mục bởi Nguyen Tang Sy (Tìm giới hạn từ 07-06-2020)



Sắp theo                Sắp xếp  

#588996 Topic Ôn thi HSG 9 2015-2016 (Hình học)

Đã gửi bởi Nguyen Tang Sy on 14-09-2015 - 21:56 trong Chuyên đề toán THCS

Bài 43:Cho tam giác $ABC$ nội tiếp $(O)$ có $AB < AC$ . Tiếp tuyến tại $A$ cắt $CB$ tại $T$. kẻ đường kính $AD, DB$ cắt $OT$ tại $E$. $CMR: AE // CD$ 




#499102 Min $D=\frac{8x^{2}+y}{4x}+y^{2...

Đã gửi bởi Nguyen Tang Sy on 14-05-2014 - 22:16 trong Bất đẳng thức và cực trị

1) Cho $x+y\geq 1;x>0$ Tìm Min $D=\frac{8x^{2}+y}{4x}+y^{2}$

Em chém bừa vậy  :icon6: :ukliam2: 

$D \geq \frac{8x^{2} + 1 - x }{4x} + y^{2} $

xét $x < 1 \rightarrow y > 0 $

do đó: $ y \geq 1- x \rightarrow y^{2} \geq (1-x)^{2}$ 

$\rightarrow  D \geq \frac{8x^{2} + 1 - x }{4x} + (1-x)^{2} $

.....

$ => min D = 1,5$

 

xét $x >= 1$ thì ta có: $y^{2} \geq 0$ . do đó:

 $D \geq  \rightarrow \frac{8x^{2} + 1 - x}{4x} $

cm đc lúc này $min D = 2$ đạt tại $ x = 1$

 

suy ra $min$ $D = 1,5$




#499166 Min $D=\frac{8x^{2}+y}{4x}+y^{2...

Đã gửi bởi Nguyen Tang Sy on 15-05-2014 - 12:03 trong Bất đẳng thức và cực trị

Chỗ nhân có vấn đề hay sao ấy?Đây nhé:
$(y-x)\geq 1-2x\Rightarrow (y-x)(y+x)\geq (1-2x)(y+x);(y+x)\geq 1\Rightarrow (1-2x)(y+x)\geq 1-2x(????????)$  Biết 1-2x âm dương ra sao?

ukm, t quên. phải xét $x < 1$ và $x >= 1$. sửa rùi đó. xem thử đc hok 




#500194 $\frac{a^2}{a+2b^3}+\frac{b^2}...

Đã gửi bởi Nguyen Tang Sy on 19-05-2014 - 22:54 trong Bất đẳng thức và cực trị

$P = \sum \frac{a^2}{a+2b^3} = \sum a - \sum \frac{2ab^3}{a+2b^3} = 3 - \sum \frac{2ab^3}{a+2b^3} $

ta có: $ \sum \frac{2ab^3}{a+b^3 + b^3} \geq \sum \frac{2ab^3}{3\sqrt[3]{ab^6}} = \sum \frac{2ab^3}{3\sqrt[3]{ab^6}} = \frac{2}{3}\sum b\sqrt[3]{a^2}$ 

Lại có:$a +  ac + ac \geq a\sqrt[3]{c^2}$

           $b + ba + ba \geq b\sqrt[3]{a^2}$

           $c + bc + bc \geq c\sqrt[3]{b^2}$

cộng theo vế ta có:

$(a+b+c) + 2(ab+bc+ca) \geq 3\sum b\sqrt[3]{a^2}$

suy ra: $3\sum b\sqrt[3]{a^2} \leq 3+2.3 = 9 $

do đó: $ \sum b\sqrt [3]{a^2} \leq 3$

từ đó $P \geq 3 - \frac{2.3}{3} = 1$




#498720 $M=x.(x^2+y)+y(y^2+x)$

Đã gửi bởi Nguyen Tang Sy on 13-05-2014 - 08:23 trong Bất đẳng thức và cực trị

<_< bác bỏ 6040 làm cảnh à 

tks bác!! để em sửa. 




#498716 $M=x.(x^2+y)+y(y^2+x)$

Đã gửi bởi Nguyen Tang Sy on 13-05-2014 - 08:12 trong Bất đẳng thức và cực trị

Ta có:

$M=x^3+y^3+2xy=(x+y)^3-3xy(x+y)+2xy=2014^3-6042xy+2xy=2014^3-6040xy$

Ta lại có:

$(x+y)^2\geq 4xy\Rightarrow xy\leq \frac{2014^2}{4}$

Do $x,y$ là các số tự nhiên nên $xy\geq 1$

Vậy ta có:

$1\le xy \le \frac{2014^2}{4}\\\Rightarrow -6040\geq -6040xy\geq -1510.2014^2\\\Rightarrow 2014^3-6040\geq 2014^3-6040xy\geq 2014^3-1510.2014^2\\\Rightarrow 2014^3-6040\ge M \ge 2014^3-1510.2014^2$

Bận bịu quá!Làm gấp nên khg chắc!Bạn tự tìm dấu bằng nhé!

sai rồi bạn!! $x + y = 2014$ mà




#498590 $P=\frac{ab+bc+ca}{a^2+b^2+c^2}+\frac...

Đã gửi bởi Nguyen Tang Sy on 12-05-2014 - 18:54 trong Bất đẳng thức và cực trị

ta có:

$\frac{(a+b+c)^{3}}{abc}$
$=\frac{a+b+c}{abc}.(a+b+c)^{2}$
$=(\frac{1}{ab}+\frac{1}{bc}+\frac{1}{ca}).(a^{2}+b^{2}+c^{2}+2ab+2bc+2ca)$
$\geq \frac{9}{ab+bc+ca}(a^{2}+b^{2}+c^{2}+2ab+2bc+2ca)$
$=18+\frac{9(a^{2}+b^{2}+c^{2})}{ab+bc+ca}$
 
do đó:
$A \geq 18 + (\frac{ab+bc+ca}{a^{2}+b^{2}+c^{2}} + \frac{a^{2}+b^{2}+c^{2}} {ab+bc+ca} )+ \frac{8(a^{2}+b^{2}+c^{2})}{ab+bc+ca}$
$\rightarrow A \geq 18 + 2 + 8 = 28$



#498714 $M=x.(x^2+y)+y(y^2+x)$

Đã gửi bởi Nguyen Tang Sy on 13-05-2014 - 08:01 trong Bất đẳng thức và cực trị

biến đổi:

$ M = x^{3} + y^{3} + 2xy = (x+y)^{3} -3xy(x+y) + 2xy = 2014^{3} -6040xy$

Tìm min:

để M $min$ thì $xy$ phải $max$. ta có $xy \leq \frac{(x+y)^{2}}{4}$

do đó min $M = 2014^{3} - \frac{6040.2014^{2}}{4}$

Tìm $max$:

M đạt max thì $xy$ phải $min$

giả sử xy đạt min . ta sẽ chứng  2 số $x,y$ không  đồng thời lớn hơn 1

thật vậy, giả sử $ y \geq x \geq 2$

ta chọn 2 số $x - 1$ và $y+1$  (vì nó có tổng bằng 2014) 

ta có  $(x-1)(y+1) > 0$ và $xy - (x-1)(y+1) = y-x + 1 > 0$

tức là ta tìm đc tích mới nhỏ hơn tích xy , trái với $xy$ min. 

vậy phải có 1 số = 1 và số còn lại bằng 2013

khi đó min $xy$ = 2013

và max $M = 2014^{3}- 6040*2013 $




#499497 $A=\frac{1+a^2}{1+b^2}+\frac{1+b^2...

Đã gửi bởi Nguyen Tang Sy on 16-05-2014 - 23:14 trong Bất đẳng thức và cực trị

4)cho a,b,c là các số thực dương thoả $a^2+b^2+c^2=5(a+b+c)-2ab$ tìm giá trị nhỏ nhất của 

$P=a+b+c+48\begin{pmatrix}\frac{\sqrt{3}}{\sqrt{a+10}}+\frac{1}{\sqrt[3]{b+c}}\end{pmatrix}$

$P=a+b+c+48\begin{pmatrix}\frac{\sqrt{3}}{\sqrt{a+10}}+\frac{1}{\sqrt[3]{b+c}}\end{pmatrix}$

áp dụng bất đẳng thức AM-GM ta có:

$a + 10 + \frac{24\sqrt{3}}{\sqrt{a+10}} + \frac{24\sqrt{3}}{\sqrt{a+10}} \geq  36$

$b + c + \frac{16}{\sqrt[3]{b+c}} + \frac{16}{\sqrt[3]{b+c}} + \frac{16}{\sqrt[3]{b+c}} \geq  32 $

cộng theo vế ta có: $P + 10 \geq 68$ suy ra: $P \geq 58$

dấu "=" xảy ra khi $a = 2 ;  b = 3 ; c = 5$ và khi đó  $a^2+b^2+c^2=5(a+b+c)-2ab$




#498580 $$A=(a+b+1)(a^2+b^2)+\frac{4}{a+b}$...

Đã gửi bởi Nguyen Tang Sy on 12-05-2014 - 18:32 trong Bất đẳng thức và cực trị

ta có: $a +b \geq 2$ và $a^{2} + b^{2} \geq 2$

$ A = (a^{2}+b^{2})(a+b)+\frac{8}{a+b} + a^{2}+b^{2}-\frac{4}{a+b} $

$(a^{2}+b^{2})(a+b)+\frac{8}{a+b}\geq 2\sqrt{8(a^{2}+b^{2})} \geq  8$

$a^{2}+b^{2}\geq 2ab = 2$

$\frac{-4}{a+b}\geq -2$

$\rightarrow A >= 8 + 2 - 2 = 8$




#518771 $\left\{\begin{matrix} x^{4}-y^...

Đã gửi bởi Nguyen Tang Sy on 10-08-2014 - 11:23 trong Phương trình, hệ phương trình và bất phương trình

Giải hệ phương trình:

 

1/

$\left\{\begin{matrix} \frac{1}{x}-\frac{1}{2y}=2(y^{4}-x^{4}) & \\ \frac{1}{x}+\frac{1}{2y}=(x^{2}+3y^{2})(3x^{2}+y^{2}) & \end{matrix}\right.$

 

Từ hệ suy ra:
$\left\{\begin{matrix} \frac{2}{x}=5y^4 + x^4 + 10x^2y^2 & \\ \frac{1}{y} = 5x^4 + y^4 + 10x^2y^2 \end{matrix}\right.$
$\Leftrightarrow \left\{\begin{matrix} 5xy^4 + x^5 + 10x^3y^2 = 2 & \\ 5x^4y + y^5 + 10x^2y^3 = 1 \end{matrix}\right.$
Lần lượt cộng trừ 2 phương trình ta có: 
$ \left\{\begin{matrix} (x+y)^5 = 3 & \\ (x-y)^5 = 1 \end{matrix}\right. $ 
:icon6:  :lol:



#518749 $\left\{\begin{matrix} x^{4}-y^...

Đã gửi bởi Nguyen Tang Sy on 10-08-2014 - 09:32 trong Phương trình, hệ phương trình và bất phương trình

2/

$\left\{\begin{matrix} x^{4}-y^{4}=240 & \\ x^{3}-2y^{3}=3(x^{2}-4y^{2})-4(x-8y) & \end{matrix}\right.$

$pt(1) - 8.pt(2) $
$\Leftrightarrow  x^4 - 8(x^3-3x^2+4x)+16 = y^4 - 8(2y^3-12y^2+32y)+256$
$\Leftrightarrow (x-2)^4 = (y-4)^4$
Tới đây là ok rồi!!  :icon6:



#500477 Tìm $Min$ của $xy + yz + zx - 2xyz$

Đã gửi bởi Nguyen Tang Sy on 21-05-2014 - 12:51 trong Bất đẳng thức và cực trị

theo mình x,y,z dương

$\frac{1}{x} + \frac{1}{y} + \frac{1}{z} \geq  \frac{9}{x+y+z} = \frac{3}{2}$

suy ra: $xy+yz+zx \geq \frac{3xyz}{2}$

       $\Rightarrow \frac{4(xy+yz+zx))}{3} \geq 2xyz$

do đó: $xy+yz+zx -2xyz \geq xy+yz+zx -  \frac{4(xy+yz+zx))}{3} =  \frac{-(xy+yz+zx)}{3} \geq \frac{-(x+y+z)^2}{9} = -4 $




#500448 Chứng tỏ rằng: $a^{3}-b^{3}+c^{3}+3abc= (a...

Đã gửi bởi Nguyen Tang Sy on 21-05-2014 - 09:11 trong Đại số

 

c, Tìm các số hữu tỉ p,q,r để có đẳng thức $\frac{3-3\sqrt[3]{4}}{1-\sqrt[3]{2}+\sqrt[3]{4}}= p+q\sqrt[3]{2}+r\sqrt[3]{4}$

đặt $x = \sqrt[3]{2}$ suy ra $x^3 = 2$

ta có:

$\frac{3-3\sqrt[3]{4}}{1-\sqrt[3]{2}+\sqrt[3]{4}}$

$=\frac{3-3x^2}{1-x+x^2}$
$= \frac{(3-3x^2)(x+1)}{x^3+1} $
$= \frac{(3-3x^2)(x+1)}{3} $
$=(1-x^2)(x+1)$
$=x+1-x^3 - x^2 $
$=-x^2 + x -1$
suy ra: p = -1 ; q = 1 ; r = -1



#499445 1.$\frac{x-\sqrt{x}}{1-\sqrt...

Đã gửi bởi Nguyen Tang Sy on 16-05-2014 - 20:20 trong Phương trình, hệ phương trình và bất phương trình

2.$\frac{1}{1-x^{2}}+1> \frac{3x}{\sqrt{1-x^{2}}}$

điều kiện: $ -1 < x < 1$

$\frac{1}{1-x^{2}}+1> \frac{3x}{\sqrt{1-x^{2}}}$
$\Leftrightarrow 2 - x^{2} > 3x\sqrt{1-x^{2}}$          $(1)$
Nếu $ -1 <  x < 0 $ thì $(1)$ luôn đúng.    $(2)$
nếu $1 > x >= 0 $ ta có:
$(1)$  <=> $  4 - 4x^{2} + x^{4} > 9x^{2}(1-x^{2}) $
$\Leftrightarrow 10x^{4} -13x^{2} +4 > 0$
$\rightarrow 0 \leq x^{2} < 0,5$ hoặc  $ 1 > x^{2} > 0,8 $
suy ra:  $0 < x < \sqrt{0,5}$ hoặc $1 > x > \sqrt{0,8}$
kết hợp với $(2)$ ta có: $ -1 < x < \sqrt{0,5}$ hoặc $1 > x > \sqrt{0,8}$



#499419 1.$\frac{x-\sqrt{x}}{1-\sqrt...

Đã gửi bởi Nguyen Tang Sy on 16-05-2014 - 19:18 trong Phương trình, hệ phương trình và bất phương trình

3.$\sqrt{3x+1}-\sqrt{6-x}+3x^{2}-14x-8=0$

 

Điều kiện: $ \frac{-1}{3} \leq x \leq 6$

$\Leftrightarrow \sqrt{3x+1}-4-\sqrt{6-x}+1+3x^{2}-14x-5=0$
$\Leftrightarrow \frac{3(x-5)}{\sqrt{3x+1}+4} + \frac{x-5}{\sqrt{6-x}+1} + (3x+1)(x-5)$
$\Leftrightarrow (x-5)(\frac{1}{\sqrt{3x+1}+4} + \frac{1}{\sqrt{6-x}+1} + 3x+1)$
 
Chú ý trong ngoặc lớn hơn 0 do đó x = 5



#498708 Chứng minh $a^{2}+b^{2}+c^{2}\geqslan...

Đã gửi bởi Nguyen Tang Sy on 13-05-2014 - 07:08 trong Bất đẳng thức và cực trị

suy nghĩ đi các bạn.!!! đợi đáp án làm gì!!

$a^{2}+b^{2}+c^{2}\geqslant \frac{9abc}{a+b+c}+\left ( c-a \right )^{2}$

$\Leftrightarrow b^{2} + 2ac \geqslant \frac{9abc}{a+b+c}$
$\Leftrightarrow (b^{2} + 2ac)(a+b+c)\geq 9abc$
ta có:  
$b^{2} + 2ac = b^2 + ac +ac \geq 3\sqrt[3]{b^{2}a^{2}c^{2}}$     $ (1) $
$a+ b +c \geq b+ 2\sqrt{ac} = b +\sqrt{ac}+\sqrt{ac}\geq 3\sqrt[3]{abc}$   $ (2) $
 
nhân 2 vế (1) và (2) ta có đpcm



#500601 $S=1^5+2^5+...+x^5$

Đã gửi bởi Nguyen Tang Sy on 21-05-2014 - 21:12 trong Đại số

xét $g(x) = x^5$

bạn tìm đa thức $f(x) = ax^6 + bx^5 + cx^4 + dx^3 + ex^2 + fx + g $sao cho: $g(x) = f(x) - f(x-1)$ (bạn khai triển ra rồi tìm các hệ số a,b,c,d,e,f,g bằng cách đồng nhất hệ số)

khi đó công thức cần tìm có dạng $S = g(1) + g(2) + ... + g(x) = f(1) - f(0) +f(2) - f(1) + f(3) - f(2) + ... + f(x) - f(x-1) = f(x) - f(0) $




#498849 Giải hệ phương trình sau: $\left\{\begin{matrix...

Đã gửi bởi Nguyen Tang Sy on 13-05-2014 - 21:11 trong Phương trình, hệ phương trình và bất phương trình

Bạn ơi mình cũng thử qua cách này rồi! Nhưng không được, hệ này thầy mình bảo ra nghiệm vô tỉ. Mình đã thử rất nhiều cách và cũng rất lâu nhưng không tìm ra. ( thầy lấy trong đề thi HSG của mấy tỉnh miền Bắc gì đó. Thầy bảo khi nào tụi mình có hướng đi thì thầy mới sửa )

hì, mình  tưởng nghiệm đẹp nên ko giải nữa  :icon10: , giờ giải lại thì thấy nghiệm vô tỉ  :( .giờ t cũng đang nghĩ đây :wacko: . khi nào thầy sửa thì nhắn cho t nhé!! t cũng muốn biết thầy giải như thế nào!!  :lol:




#498527 Giải hệ phương trình sau: $\left\{\begin{matrix...

Đã gửi bởi Nguyen Tang Sy on 12-05-2014 - 15:02 trong Phương trình, hệ phương trình và bất phương trình

 
$pt (1) \Leftrightarrow x^{2}-x = 19-y^{2}$
$pt(2) \Leftrightarrow  x(x-1)= \frac{20}{y(2-y)}$
  $    \Leftrightarrow x^{2}-x = \frac{20}{y(2-y)}$
$suy ra: 19-y^{2} = \frac{20}{y(2-y)}$
 
đến đây bạn tự giải tiếp nhé!!



#498298 Trích đề thi

Đã gửi bởi Nguyen Tang Sy on 10-05-2014 - 22:18 trong Tài liệu - Đề thi

Đặt $  a = x- y   (a < 2)  và b = xy $

Biến đổi phương trình thành:

$ (a-2)[(a^{2} + 3b)(a-3) + 3b] = 0 $
$ \Rightarrow  (a^{2}+3b)(a-3) + 3b = 0 $
$ \Leftrightarrow  (a^{2} + 3b)(a-2) = a^{2} $
Chú ý rẳng VT <= 0 từ đó a = 0 và x = y = 0;



#500402 $\left\{\begin{matrix} x^2+y^2+xy+1=4y...

Đã gửi bởi Nguyen Tang Sy on 20-05-2014 - 22:24 trong Phương trình, hệ phương trình và bất phương trình

Vì $y = 0$ ko là nghiệm của hệ nên chia 2 phương trình của hệ cho $y$ ta có:

$\left\{\begin{matrix} \frac{x^2+1}{y} + x +y = 4\\ \frac{x^2+1}{y}.(x+y-2)=1 \end{matrix}\right.$

tới đây đặt $a = \frac{x^2+1}{y} $ và $b = x+y-2$ :icon6:

hệ trở thành:

$\left\{\begin{matrix} a + b = 2\\ ab =1 \end{matrix}\right.$
suy ra: $ a = b = 1$
$\left\{\begin{matrix} \frac{x^2+1}{y} = 1\\ x+y-2 = 1 \end{matrix}\right.$
$\Leftrightarrow \left\{\begin{matrix} x^2+1 = y\\ x  = 3 - y\end{matrix}\right.$
$\Leftrightarrow \left\{\begin{matrix} x = 3 - y\\  (3-y)^2 + 1 = y\end{matrix}\right.$
 
:wub:



#500106 1. $\left\{\begin{matrix} (\frac{x}{y}+\frac{y}...

Đã gửi bởi Nguyen Tang Sy on 19-05-2014 - 17:52 trong Phương trình - hệ phương trình - bất phương trình

 Giải hpt:

1. $\left\{\begin{matrix} (\frac{x}{y}+\frac{y}{x})(x+y)=15\\(\frac{x^{2}}{y^{2}}+\frac{y^{2}}{x^{2}})(x^{2}+y^{2})=85 \end{matrix}\right.$

 

2. $\left\{\begin{matrix} 2x^{2}+x-\frac{1}{y}=2\\ y-y^{2}x-2y^{2}=-2 \end{matrix}\right.$

 

3. $\left\{\begin{matrix} x\sqrt{y-1}+y\sqrt{x-1}=3\\ 6\frac{(x+y)^{2}}{xy}+x^{2}+y^{2}-5(x+y)=\frac{2x^{2}}{y}+\frac{3y^{2}}{x}+6 \end{matrix}\right.$

 

Mod: Chú ý cách đặt tiêu đề và gõ latex

đặt $a = \frac{1}{y} $   :icon6:
hpt trở thành:
$ \left\{\begin{matrix} 2x^{2}+x-a=2\\ \frac{1}{a}-\frac{x}{a^2}-\frac{2}{a^2}= -2 \end{matrix}\right. $
$\Leftrightarrow $ $\left\{\begin{matrix} 2x^{2}+x-a=2\\ \ a-x- 2 = -2a^2 \end{matrix}\right.$
$\Leftrightarrow $ $\left\{\begin{matrix} 2x^{2}+x-a=2\\ \ 2a^2+a - x = 2 \end{matrix}\right.$
tới đây bạn trừ 2 pt cho nhau ta đc:
$(a-x)(2a+2x+2) = 0$  :icon10:
tới đây thế lại vào hệ là OK!!!  :wub:



#499455 CMR: $(x-y)^{3}+(y-z)^{3}+(z-x)^{3}$...

Đã gửi bởi Nguyen Tang Sy on 16-05-2014 - 20:45 trong Số học

  $x+y+z=(x-y)(y-z)(z-x)$   $(1)$  

Nếu 3 số x , y, z có số dư khác nhau khi chia cho 3 thì x -y ,y - z , z -x cùng ko chia hết cho 3

Mà x + y + z chia hết cho 3 . từ (1) suy ra vô lí.

+ Nếu trong 3 sô x,y,z chỉ có 2 số chia cho 3 có cùng số dư thì 1 trong 3 hiệu x -y ,y - z , z -x có 1 hiệu chia hết cho 3

mà x + y + z ko chia hết cho 3 nên từ (1) suy ra vô lí.

vậy x,y,z có cùng số dư khi chia cho 3. do dó  (x -y) (y - z) ( z -x ) chia hết cho 27

đặt a = x -y.  b = y-z .  c = z-x thì a+b+c = 0 và abc chia hết cho 27

suy ra $a^{3} + b^{3} + c^{3} = 3abc$ chia hết cho 3.27 = 81

 

P/S: bị chậm mất rồi :(




#500426 Tìm giá trị nhỏ nhất của biểu thức: $\frac{x^2+y^2-1}{(1-x)(1-y)}...

Đã gửi bởi Nguyen Tang Sy on 21-05-2014 - 07:12 trong Bất đẳng thức và cực trị

$\frac{x^2+y^2-1}{(1-x)(1-y)} \geq  \frac{\frac{(x+y)^2}{2}-1}{\frac{(2-x-y)^2}{4}} = \frac{2[(x+y)^2-1]}{(2-x-y)^2}$

xét hàm $f(t) = \frac{t^2-1}{(2-t)^2}$ với $ t \geq 0$

chứng minh được $f(t) \geq f(\frac{1}{2}) $